You are on page 1of 26

Chapter 43: Assessment of Digestive and 6.

A patient has come to the outpatient radiology


Gastrointestinal Function department for diagnostic testing. Which of the following
diagnostic procedures will allow the care team to evaluate
1. A nurse is caring for a patient who is scheduled for a
and remove polyps?
colonoscopy and whose bowel preparation will include
A) Colonoscopy
polyethylene glycol electrolyte lavage prior to the
B) Barium enema
procedure. The presence of what health problem would
C) ERCP
contraindicate the use of this form of bowel preparation?
D) Upper gastrointestinal fibroscopy
A) Inflammatory bowel disease
B) Intestinal polyps
7. A nurse is caring for a patient with recurrent
C) Diverticulitis
hematemesis who is scheduled for upper gastrointestinal
D) Colon cancer
fibroscopy (UGF). How should the nurse in the radiology
department prepare this patient?
2. A nurse is promoting increased protein intake to
A) Insert a nasogastric tube.
enhance a patients wound healing. The nurse knows
B) Administer a micro Fleet enema at least 3 hours before
that enzymes are essential in the digestion of nutrients
the procedure.
such as protein. What is the enzyme that initiates
C) Have the patient lie in a supine position for the
the digestion of protein?
procedure.
A) Pepsin
D) Apply local anesthetic to the back of the patients
B) Intrinsic factor
throat.
C) Lipase
D) Amylase
8. The nurse is providing health education to a patient
scheduled for a colonoscopy. The nurse should explain
3. A patient has been brought to the emergency
that she will be placed in what position during this
department with abdominal pain and is subsequently
diagnostic test?
diagnosed with appendicitis. The patient is scheduled for
A) In a knee-chest position (lithotomy position)
an appendectomy but questions the nurse about how his
B) Lying prone with legs drawn toward the chest
health will be affected by the absence of an appendix.
C) Lying on the left side with legs drawn toward the chest
How should the nurse best respond?
D) In a prone position with two pillows elevating the
A) Your appendix doesnt play a major role, so you wont
buttocks
notice any difference after you recovery from surgery.
B) The surgeon will encourage you to limit your fat intake
9. A patient has sought care because of recent dark-
for a few weeks after the surgery, but your body will then
colored stools. As a result, a fecal occult blood test has
begin to compensate.
been ordered. The nurse should instruct the patient to
C) Your body will absorb slightly fewer nutrients from the
avoid which of the following prior to collecting a
food you eat, but you wont be aware of this.
stool sample?
D) Your large intestine will adapt over time to the
A) NSAIDs
absence of your appendix.
B) Acetaminophen
C) OTC vitamin D supplements
4. A patient asks the nursing assistant for a bedpan. When
D) Fiber supplements
the patient is finished, the nursing assistant notifies the
nurse that the patient has bright red streaking of blood in
10. The nurse is preparing to perform a patients
the stool. What is this most likely a result of?
abdominal assessment. What examination sequence
A) Diet high in red meat
should the nurse follow?
B) Upper GI bleed
A) Inspection, auscultation, percussion, and palpation
C) Hemorrhoids
B) Inspection, palpation, auscultation, and percussion
D) Use of iron supplements
C) Inspection, percussion, palpation, and auscultation
5. An adult patient is scheduled for an upper GI series D) Inspection, palpation, percussion, and auscultation
that will use a barium swallow. What teaching should the
nurse include when the patient has completed the test? 11. A patient who has been experiencing changes in his
A) Stool will be yellow for the first 24 hours bowel function is scheduled for a barium enema.
postprocedure. What instruction should the nurse provide for
B) The barium may cause diarrhea for the next 24 hours. postprocedure recovery?
C) Fluids must be increased to facilitate the evacuation of A) Remain NPO for 6 hours postprocedure.
the stool. B) Administer a Fleet enema to cleanse the bowel of the
D) Slight anal bleeding may be noted as the barium is barium.
passed. C) Increase fluid intake to evacuate the barium.
D) Avoid dairy products for 24 hours postprocedure.

1
12. A nurse is caring for a newly admitted patient with a C) The presence of fat in the patients stool
suspected GI bleed. The nurse assesses the patients stool D) Persistently low hemoglobin and hematocrit
after a bowel movement and notes it to be a tarry-black
color. This finding is suggestive of bleeding from what 19. A patient with a recent history of intermittent bleeding
location? is undergoing capsule endoscopy to determine the source
A) Sigmoid colon of the bleeding. When explaining this diagnostic test to
B) Upper GI tract the patient, what advantage should the nurse describe?
C) Large intestine A) The test allows visualization of the entire peritoneal
D) Anus or rectum cavity.
B) The test allows for painless biopsy collection.
13. A nursing student has auscultated a patients abdomen C) The test does not require fasting.
and noted one or two bowel sounds in a 2-minute period D) The test is noninvasive.
of time. How would you tell the student to document the
patients bowel sounds? 20. A nurse is caring for a patient admitted with a
A) Normal suspected malabsorption disorder. The nurse knows that
B) Hypoactive one of the accessory organs of the digestive system is the
C) Hyperactive pancreas. What digestive enzymes does the pancreas
D) Paralytic ileus secrete? Select all that apply.
A) Pepsin
14. An advanced practice nurse is assessing the size and B) Lipase
density of a patients abdominal organs. If the results of C) Amylase
palpation are unclear to the nurse, what assessment D) Trypsin
technique should be implemented? E) Ptyalin
A) Percussion
B) Auscultation 21. The nurse is caring for a patient with a duodenal ulcer
C) Inspection and is relating the patients symptoms to the physiologic
D) Rectal examination functions of the small intestine. What do these functions
include? Select all that apply.
15. A nurse is caring for a patient with biliary colic and is A) Secretion of hydrochloric acid (HCl)
aware that the patient may experience referred B) Reabsorption of water
abdominal pain. Where would the nurse most likely C)Secretion of mucus
expect this patient to experience referred pain? D) Absorption of nutrients
A) Midline near the umbilicus E) Movement of nutrients into the bloodstream
B) Below the right nipple
C) Left groin area 22. A nurse is performing an abdominal assessment of an
D) Right lower abdominal quadrant older adult patient. When collecting and analyzing data,
. the nurse should be cognizant of what age-related change
16. An inpatient has returned to the medical unit after a in gastrointestinal structure and function?
barium enema. When assessing the patients A) Increased gastric motility
subsequent bowel patterns and stools, what finding B) Decreased gastric pH
should the nurse report to the physician? C) Increased gag reflex
A) Large, wide stools D) Decreased mucus secretion
B) Milky white stools
C) Three stools during an 8-hour period of time 23. The nurse educator is reviewing the blood supply of
D) Streaks of blood present in the stool the GI tract with a group of medical nurses. The nurse is
explaining the fact that the veins that return blood from
17. A nurse in a stroke rehabilitation facility recognizes the digestive organs and the spleen form the portal
that the brain regulates swallowing. Damage to what venous system. What large veins will the nurse list when
area of the brain will most affect the patients ability to describing this system? Select all that apply.
swallow? A) Splenic vein
A) Temporal lobe B) Inferior mesenteric vein
B) Medulla oblongata C) Gastric vein
C) Cerebellum D) Inferior vena cava
D) Pons

18. A patient is being assessed for a suspected deficit in


intrinsic factor synthesis. What diagnostic or assessment
finding is the most likely rationale for this examination of
intrinsic factor production?
A) Muscle wasting
B) Chronic jaundice in the absence of liver disease
2
24. The physiology instructor is discussing the GI system 29. A patient will be undergoing abdominal computed
with the pre-nursing class. What should the instructor tomography (CT) with contrast. The nurse has
describe as a major function of the GI tract? administered IV sodium bicarbonate and oral
A) The breakdown of food particles into cell form for acetylcysteine (Mucomyst) before the study as ordered.
digestion What would indicate that these medications have had the
B) The maintenance of fluid and acid-base balance desired therapeutic effect?
C) The absorption into the bloodstream of nutrient A) The patients BUN and creatinine levels are within
molecules produced by digestion reference range following the CT.
D) The control of absorption and elimination of B) The CT yields high-quality images.
electrolytes C) The patients electrolytes are stable in the 48 hours
following the CT.
25. A nurse is providing preprocedure education for a D) The patients intake and output are in balance on the
patient who will undergo a lower GI tract study the day after the CT.
following week. What should the nurse teach the patient 30. A medical patients CA 19-9 levels have become
about bowel preparation? available and they are significantly elevated. How
A) You'll need to fast for at least 18 hours prior to your should the nurse best interpret this diagnostic finding?
test. A) The patient may have cancer, but other GI disease must
B) Starting today, take over-the-counter stool softeners be ruled out.
twice daily. B) The patient most likely has early-stage colorectal
C) You'll need to have enemas the day before the test. cancer.
D) For 24 hours before the test, insert a glycerin C) The patient has a genetic predisposition to gastric
suppository every 4 hours. cancer.
D) The patient has cancer, but the site is unknown.
26. A patient presents at the walk-in clinic complaining of
recurrent sharp stomach pain that is relieved by eating. 31. A patient has come to the clinic complaining of blood
The nurse suspects that the patient may have an ulcer. in his stool. A FOBT test is performed but is negative.
How would the nurse explain the formation and role of Based on the patients history, the physician suggests a
acid in the stomach to the patient? colonoscopy, but the patient refuses, citing a strong
A) Hydrochloric acid is secreted by glands in the stomach aversion to the invasive nature of the test. What other
in response to the actual or anticipated presence of food. test might the physician order to check for blood in the
B) As digestion occurs in the stomach, the stomach stool?
combines free hydrogen ions from the food to A) A laparoscopic intestinal mucosa biopsy
form acid. B) A quantitative fecal immunochemical test
C) The body requires an acidic environment in order to C) Computed tomography (CT)
synthesize pancreatic digestive enzymes; the stomach D) Magnetic resonance imagery (MRI)
provides this environment.
D) The acidic environment in the stomach exists to buffer 32. A nurse is assessing the abdomen of a patient just
the highly alkaline environment in the esophagus. admitted to the unit with a suspected GI disease.
Inspection reveals several diverse lesions on the patients
27. Results of a patients preliminary assessment abdomen. How should the nurse best interpret this
prompted an examination of the patients assessment finding?
carcinoembryonic antigen (CEA) levels, which have come A) Abdominal lesions are usually due to age-related skin
back positive. What is the nurses most appropriate changes.
response to this finding? B) Integumentary diseases often cause GI disorders.
A) Perform a focused abdominal assessment. C) GI diseases often produce skin changes.
B) Prepare to meet the patients psychosocial needs. D) The patient needs to be assessed for self-harm.
C) Liaise with the nurse practitioner to perform an
anorectal examination. 33.Probably the most widely used in-office or at-home
D) Encourage the patient to adhere to recommended occult blood test is the Hemoccult II. The patient has come
screening protocols. to the clinic because he thinks there is blood in his stool.
When you reviewed his medications, you noted he is on
28. A clinic patient has described recent dark-colored antihypertensive drugs and NSAIDs for early arthritic pain.
stools;the nurse recognizes the need for fecal occult You are sending the patient home with the supplies
blood testing (FOBT). What aspect of the patients current necessary to perform 2 hemoccult tests on his stool and
health status would contraindicate FOBT? mail the samples back to the clinic. What instruction
A) Gastroesophageal reflux disease (GERD) would you give this patient?
B) Peptic ulcers A) Take all your medications as usual.
C) Hemorrhoids B) Take all your medications except the antihypertensive
D) Recurrent nausea and vomiting medications.
C) Dont eat highly acidic foods 72 hours before you start
the test.
3
D) Avoid vitamin C for 72 hours before you start the test. 40. A female patient has presented to the emergency
34. A patients sigmoidoscopy has been successfully department with right upper quadrant pain; the
completed and the patient is preparing to return home. physician has ordered abdominal ultrasound to rule out
Which of the following teaching points should the nurse cholecystitis (gallbladder infection). The patient
include in the patients discharge education? expresses concern to the nurse about the safety of this
A) The patient should drink at least 2 liters of fluid in the diagnostic procedure. How should the nurse best
next 12 hours. respond?
B) The patient can resume a normal routine immediately. A) Abdominal ultrasound is very safe, but it cant be
C) The patient should expect fecal urgency for several performed if youre pregnant.
hours. B) Abdominal ultrasound poses no known safety risks of
D) The patient can expect some scant rectal bleeding. any kind.
C) Current guidelines state that a person can have up to 3
35. A nurse is caring for an 83-year-old patient who is ultrasounds per year.
being assessed for recurrent and intractable nausea. D) Current guidelines state that a person can have up to 6
What age-related change to the GI system may be a ultrasounds per year.
contributor to the patients health complaint?
A) Stomach emptying takes place more slowly.
Chapter 44: Digestive and Gastrointestinal Treatment
B) The villi and epithelium of the small intestine become
thinner. Modalities
C) The esophageal sphincter becomes incompetent. 1. A nurse is preparing to place a patients ordered
D) Saliva production decreases. nasogastric tube. How should the nurse best determine
the correct length of the nasogastric tube?
36. A patient has been experiencing significant A) Place distal tip to nose, then ear tip and end of xiphoid
psychosocial stress in recent weeks. The nurse is aware of process.
the hormonal effects of stress, including norepinephrine B) Instruct the patient to lie prone and measure tip of
release. Release of this substance would have what effect nose to umbilical area.
on the patients gastrointestinal function? C) Insert the tube into the patients nose until secretions
Select all that apply. can be aspirated.
A) Decreased motility D) Obtain an order from the physician for the length of
B) Increased sphincter tone tube to insert.
C) Increased enzyme release
D) Inhibition of secretions 2. A patient is concerned about leakage of gastric contents
E) Increased peristalsis out of the gastric sump tube the nurse has just inserted.
What would the nurse do to prevent reflux gastric
37. A patient with cystic fibrosis takes pancreatic enzyme contents from coming through the blue vent of a gastric
replacements on a regular basis. The patients intake of sump tube?
trypsin facilitates what aspect of GI function? A) Prime the tubing with 20 mL of normal saline.
A) Vitamin D synthesis B) Keep the vent lumen above the patients waist.
B) Digestion of fats C) Maintain the patient in a high Fowlers position.
C) Maintenance of peristalsis D) Have the patient pin the tube to the thigh.
D) Digestion of proteins
3. A patient receiving tube feedings is experiencing
38. The nurse is caring for a patient who has a diagnosis diarrhea. The nurse and the physician suspect that the
of AIDS. Inspection of the patients mouth reveals the new patient is experiencing dumping syndrome. What
presence of white lesions on the patients oral mucosa. intervention is most appropriate?
What is the nurses most appropriate response? A) Stop the tube feed and aspirate stomach contents.
A) Encourage the patient to gargle with salt water twice B) Increase the hourly feed rate so it finishes earlier.
daily. C) Dilute the concentration of the feeding solution.
B) Attempt to remove the lesions with a tongue D) Administer fluid replacement by IV.
depressor.
C) Make a referral to the units dietitian. 4. A nurse is admitting a patient to the postsurgical unit
D) Inform the primary care provider of this finding. following a gastrostomy. When planning assessments, the
nurse should be aware of what potential postoperative
39. A patient has been scheduled for a urea breath test in complication of a gastrostomy?
one months time. What nursing diagnosis most likely A) Premature removal of the G tube
prompted this diagnostic test? B) Bowel perforation
A) Impaired Dentition Related to Gingivitis C) Constipation
B) Risk For Impaired Skin Integrity Related to Peptic Ulcers D) Development of peptic ulcer disease (PUD)
C) Imbalanced Nutrition: Less Than Body Requirements
Related to Enzyme Deficiency
D) Diarrhea Related to Clostridium Difficile Infection
4
B) TNA can be administered over 8 hours, while PN
5. A nursing educator is reviewing the care of patients requires 24-hour administration.
with feeding tubes and endotracheal tubes (ET). The C) TNA is less costly than PN.
educator has emphasized the need to check for tube D) TNA does not require the use of a micron filter.
placement in the stomach as well as residual volume.
What is the main purpose of this nursing action? 10. A nurse is initiating parenteral nutrition (PN) to a
A) Prevent gastric ulcers postoperative patient who has developed complications.
B) Prevent aspiration The nurse should initiate therapy by performing which of
C) Prevent abdominal distention the following actions?
D) Prevent diarrhea A) Starting with a rapid infusion rate to meet the patients
nutritional needs as quickly as possible
6.The nurse is administering total parenteral nutrition B) Initiating the infusion slowly and monitoring the
(TPN) to a client who underwent surgery for gastric patients fluid and glucose tolerance
cancer. Which of the nurses assessments most directly C) Changing the rate of administration every 2 hours
addresses a major complication of TPN? based on serum electrolyte values
A) Checking the patients capillary blood glucose levels D) Increasing the rate of infusion at mealtimes to mimic
regularly the circadian rhythm of the body
B) Having the patient frequently rate his or her hunger on
a 10-point scale 11. A patients physician has determined that for the next 3
C) Measuring the patients heart rhythm at least every 6 to 4 weeks the patient will require parenteral nutrition
hours (PN). The nurse should anticipate the placement of what
D) Monitoring the patients level of consciousness each type of venous access device?
shift A) Peripheral catheter
B) Nontunneled central catheter
7. A critical care nurse is caring for a patient diagnosed C) Implantable port
with acute pancreatitis. The nurse knows that the D) Tunneled central catheter
indications for starting parenteral nutrition (PN) for this
patient are what? 12. A nurse is caring for a patient who has an order to
A) 5% deficit in body weight compared to preillness discontinue the administration of parenteral nutrition.
weight and increased caloric need What should the nurse do to prevent the occurrence of
B) Calorie deficit and muscle wasting combined with low rebound hypoglycemia in the patient?
electrolyte levels A) Administer an isotonic dextrose solution for 1 to 2
C) Inability to take in adequate oral food or fluids within 7 hours after discontinuing the PN.
days B) Administer a hypertonic dextrose solution for 1 to 2
D) Significant risk of aspiration coupled with decreased hours after discontinuing the PN.
level of consciousness C) Administer 3 ampules of dextrose 50% immediately
prior to discontinuing the PN.
8. A nurse is preparing to administer a patients D) Administer 3 ampules of dextrose 50% 1 hour after
intravenous fat emulsion simultaneously with parenteral discontinuing the PN.
nutrition (PN). Which of the following principles should
guide the nurses action? 13. A nurse is caring for a patient with a subclavian central
A) Intravenous fat emulsions may be infused line who is receiving parenteral nutrition (PN). In
simultaneously with PN through a Y-connector close to preparing a care plan for this patient, what nursing
the infusion site and should not be filtered. diagnosis should the nurse prioritize?
B) The nurse should prepare for placement of another A) Risk for Activity Intolerance Related to the Presence of
intravenous line, as intravenous fat emulsions a Subclavian Catheter
may not be infused simultaneously through the line used B) Risk for Infection Related to the Presence of a
for PN. Subclavian Catheter
C) Intravenous fat emulsions may be infused C) Risk for Functional Urinary Incontinence Related to the
simultaneously with PN through a Y-connector close to Presence of a Subclavian Catheter
the infusion site after running the emulsion through a D) Risk for Sleep Deprivation Related to the presence of a
filter. Subclavian Catheter
D) The intravenous fat emulsions can be piggy-backed
into any existing IV solution that is infusing. 14. A patients health decline necessitates the use of total
parenteral nutrition. The patient has questioned the
9. A nurse is participating in a patients care conference need for insertion of a central venous catheter, expressing
and the team is deciding between parenteral nutrition a preference for a normal IV. The nurse should know that
(PN) and a total nutritional admixture (TNA). What peripheral administration of high-concentration PN
advantages are associated with providing TNA rather than formulas is contraindicated because of the risk for what
PN? complication?
A) TNA can be mixed by a certified registered nurse. A) Chemical phlebitis
5
15. A nurse is providing care for a patient with a diagnosis 20. A patients new onset of dysphagia has required
of late-stage Alzheimers disease. The patient has just insertion of an NG tube for feeding; the nurse has
returned to the medical unit to begin supplemental modified the patients care plan accordingly. What
feedings through an NG tube. Which of the nurses intervention should the nurse include in the patients
assessments addresses this patients most significant plan of care?
potential complication of feeding? A) Confirm placement of the tube prior to each medication
A) Frequent assessment of the patients abdominal girth administration.
B) Assessment for hemorrhage from the nasal insertion B) Have the patient sip cool water to stimulate saliva
site production.
C) Frequent lung auscultation C) Keep the patient in a low Fowlers position when at rest.
D) Vigilant monitoring of the frequency and character of D) Connect the tube to continuous wall suction when not
bowel movements in use.

16. The management of the patients gastrostomy is an 21. A patient has been brought to the emergency
assessment priority for the home care nurse. What department by EMS after telling a family member that he
statement would indicate that the patient is managing deliberately took an overdose of NSAIDs a few minutes
the tube correctly? earlier. If lavage is ordered, the nurse should prepare to
A) I clean my stoma twice a day with alcohol. assist with the insertion of what type of tube?
B) The only time I flush my tube is when Im putting in A) Nasogastric tube
medications. B) Levin tube
C) I flush my tube with water before and after each of my C) Gastric sump
medications. D) Orogastric tube
D) I try to stay still most of the time to avoid dislodging
my tube. 22. A patients NG tube has become clogged after the
nurse instilled a medication that was insufficiently
17. A nurse is caring for a patient with a nasogastric tube crushed. The nurse has attempted to aspirate with a large-
for feeding. During shift assessment, the nurse bore syringe, with no success. What should the nurse do
auscultates a new onset of bilateral lung crackles and next?
notes a respiratory rate of 30 breaths per minute. A) Withdraw the NG tube 3 to 5 cm and reattempt
The patients oxygen saturation is 89% by pulse oximetry. aspiration.
After ensuring the patients immediate safety, B) Attach a syringe filled with warm water and attempt an
what is the nurses most appropriate action? in-and-out motion of instilling and aspirating.
A) Perform chest physiotherapy. C) Withdraw the NG tube slightly and attempt to dislodge
B) Reduce the height of the patients bed and remove the by flicking the tube with the fingers.
NG tube. D) Remove the NG tube promptly and obtain an order for
C) Liaise with the dietitian to obtain a feeding solution reinsertion from the primary care provider.
with lower osmolarity.
D) Report possible signs of aspiration pneumonia to the 23.
primary care provider. A nurse has obtained an order to remove a patients NG
tube and has prepared the patient accordingly. After
18. A nurse is creating a care plan for a patient with a flushing the tube and removing the nasal tape, the nurse
nasogastric tube. How should the nurse direct other attempts removal but is met with resistance. Because the
members of the care team to check correct placement of nurse is unable to overcome this resistance, what is the
the tube? most appropriate action?
A) Auscultate the patients abdomen after injecting air A) Gently twist the tube before pulling.
through the tube. B) Instill a digestive enzyme solution and reattempt
B) Assess the color and pH of aspirate. removal in 10 to 15 minutes.
C) Locate the marking made after the initial x-ray C) Flush the tube with hot tap water and reattempt
confirming placement. removal.
D) Use a combination of at least two accepted methods D) Report this finding to the patients primary care
for confirming placement. provider.

19. The nurse is assessing placement of a nasogastric tube 24. A nurse is writing a care plan for a patient with a
that the patient has had in place for 2 days. The tube is nasogastric tube in place for gastric decompression.
draining green aspirate. What is the nurses most What risk nursing diagnosis is the most appropriate
appropriate action? component of the care plan?
A) Inform the physician that the tube may be in the A) Risk for Excess Fluid Volume Related to Enteral
patients pleural space. Feedings
B) Withdraw the tube 2 to 4 cm. B) Risk for Impaired Skin Integrity Related to the Presence
C) Leave the tube in its present position. of NG Tube
D) Advance the tube up to 8 cm.
6
C) Risk for Unstable Blood Glucose Related to Enteral risk for incorrect placement?
Feedings A) The patient is obese and has a short neck.
25. A patients enteral feedings have been determined to B) The patient is agitated.
be too concentrated based on the patients development C) The patient has a history of gastroesophageal reflux
of dumping syndrome. What physiologic phenomenon disease (GERD).
caused this patients complication of enteral feeding? D) The patient is being treated for pneumonia.
A) Increased gastric secretion of HCl and gastrin because
of high osmolality of feeds 31. Prior to a patients scheduled jejunostomy, the nurse is
B) Entry of large amounts of water into the small intestine performing the preoperative assessment. What goal
because of osmotic pressure should the nurse prioritize during the preoperative
C) Mucosal irritation of the stomach and small intestine assessment?
by the high concentration of the feed A) Determining the patients nutritional needs
D) Acidbase imbalance resulting from the high volume of B) Determining that the patient fully understands the
solutes in the feed postoperative care required
C) Determining the patients ability to understand and
26. A nurse is creating a care plan for a patient who is cooperate with the procedure
receiving parenteral nutrition. The patients care plan D) Determining the patients ability to cope with an altered
should include nursing actions relevant to what potential body image
complications? Select all that apply.
A) Dumping syndrome 32. You are caring for a patient who was admitted to have
B) Clotted or displaced catheter a low-profile gastrostomy device (LPGD) placed. How soon
C) Pneumothorax after the original gastrostomy tube placement can an
D) Hyperglycemia LPGD be placed?
E) Line sepsis A) 2 weeks
B) 4 to 6 weeks
27. A nurse is caring for a patient who has a C) 2 to 3 months
gastrointestinal tube in place. Which of the following are
indications for gastrointestinal intubation? Select all that 33. A nurse is caring for a patient who is receiving
apply. parenteral nutrition. When writing this patients plan of
A) To remove gas from the stomach care, which of the following nursing diagnoses should be
B) To administer clotting factors to treat a GI bleed included?
C) To remove toxins from the stomach A) Risk for Peripheral Neurovascular Dysfunction Related
D) To open sphincters that are closed to Catheter Placement
E) To diagnose GI motility disorders B) Ineffective Role Performance Related to Parenteral
Nutrition
28. A patient with dysphagia is scheduled for PEG tube C) Bowel Incontinence Related to Parenteral Nutrition
insertion and asks the nurse how the tube will stay D) Chronic Pain Related to Catheter Placement
in place. What is the nurses best response?
A) Adhesive holds a flange in place against the abdominal 34. A nurse is aware of the high incidence of catheter-
skin. related bloodstream infections in patients receiving
B) A stitch holds the tube in place externally. parenteral nutrition. What nursing action has the greatest
C) The tube is stitched to the abdominal skin externally potential to reduce catheter-related bloodstream
and the stomach wall internally. infections?
D) An internal retention disc secures the tube against the A) Use clean technique and wear a mask during dressing
stomach wall. changes.
B) Change the dressing no more than weekly.
29. A patient is postoperative day 1 following C) Apply antibiotic ointment around the site with each
gastrostomy. The nurse is planning interventions to dressing change.
address the nursing diagnosis of Risk for Infection Related D) Irrigate the insertion site with sterile water during each
to Presence of Wound and Tube. What intervention is dressing change.
most appropriate?
A) Administer antibiotics via the tube as ordered. 35. A patient who suffered a stroke had an NG tube
B) Wash the area around the tube with soap and water inserted to facilitate feeding shortly after admission. The
daily. patient has since become comatose and the patients
C) Cleanse the skin within 2 cm of the insertion site with family asks the nurse why the physician is recommending
hydrogen peroxide once per shift. the removal of the patients NG tube and the insertion of a
D) Irrigate the skin surrounding the insertion site with gastrostomy tube. What is the nurses best response?
normal saline before each use. A) It eliminates the risk for infection.
B) Feeds can be infused at a faster rate.
30. The nurse is preparing to insert a patients ordered NG C) Regurgitation and aspiration are less likely.
tube. What factor should the nurse recognize as a
7
D) It allows caregivers to provide personal hygiene more C) Wiping the teeth and gums clean with a gauze pad
easily. D) Brushing the patients teeth with a toothbrush and small
amount of toothpaste
36. A patient has been discharged home on parenteral 2. An elderly patient comes into the emergency
nutrition (PN). Much of the nurses discharge education department complaining of an earache. The patient and
focused on coping. What must a patient on PN likely learn has an oral temperature of 100.2F and otoscopic
to cope with? Select all that apply. assessment of the ear reveals a pearly gray tympanic
A) Changes in lifestyle membrane with no evidence of discharge or inflammation.
B) Loss of eating as a social behavior Which action should the triage nurse take next?
C) Chronic bowel incontinence from GI changes A) Palpate the patients parotid glands to detect swelling
D) Sleep disturbances related to frequent urination during and tenderness.
nighttime infusions B) Assess the temporomandibular joint for evidence of a
E) Stress of choosing the correct PN formulation malocclusion.
C) Test the integrity of cranial nerve XII by asking the
37. A patient has a gastrostomy tube that has been patient to protrude the tongue.
placed to drain stomach contents by low intermittent D) Inspect the patients gums for bleeding and
suction. What is the nurses priority during this aspect of hyperpigmentation.
the patients care?
A) Measure and record drainage. 3. A patient who had a hemiglossectomy earlier in the day
B) Monitor drainage for change in color. is assessed postoperatively, revealing a patent airway,
C) Titrate the suction every hour. stable vital signs, and no bleeding or drainage from the
D) Feed the patient via the G tube as ordered. operative site. The nurse notes the patient is alert. What is
the patients priority need at this time?
38. A nurse is preparing to discharge a patient home on A) Emotional support from visitors and staff
parenteral nutrition. What should an effective home B) An effective means of communicating with the nurse
care teaching program address? Select all that apply. C) Referral to a speech therapist
A) Preparing the patient to troubleshoot for problems D) Dietary teaching focused on consistency of food and
B) Teaching the patient and family strict aseptic technique frequency of feedings
C) Teaching the patient and family how to set up the
infusion 4. The nurse notes that a patient who has undergone skin,
D) Teaching the patient to flush the line with sterile water tissue, and muscle grafting following a modified radical
E) Teaching the patient when it is safe to leave the access neck dissection requires suctioning. What is the most
site open to air important consideration for the nurse when
suctioning this patient?
39. The nurse is caring for a patient who is postoperative A) Avoid applying suction on or near the suture line.
from having a gastrostomy tube placed. What should the B) Position patient on the non operative side with the
nurse do on a daily basis to prevent skin breakdown? head of the bed down.
A) Verify tube placement. C) Assess the patients ability to perform self-suctioning.
B) Loop adhesive tape around the tube and connect it D) Evaluate the patients ability to swallow saliva and clear
securely to the abdomen. fluids.
C) Gently rotate the tube.
D) Change the wet-to-dry dressing. 5. A patient with gastroesophageal reflux disease (GERD)
has a diagnosis of Barretts esophagus with minor cell
40. A nurse is preparing to administer a patients changes. Which of the following principles should be
scheduled parenteral nutrition (PN). Upon inspecting the integrated into the patients subsequent care?
bag, the nurse notices that the presence of small amounts A) The patient will require an upper endoscopy every 6
of white precipitate are present in the bag What is the months to detect malignant changes.
nurses best action? B) Liver enzymes must be checked regularly, as H2
A) Recognize this as an expected finding. receptor antagonists may cause hepatic damage.
B) Place the bag in a warm environment for 30 minutes. C) Small amounts of blood are likely to be present in the
C) Shake the bag vigorously for 10 to 20 seconds. stools and are not cause for concern.
D)Contact the pharmacy to obtain a new bag of PN D) Antacids may be discontinued when symptoms of
heartburn subside.
Chapter 45: Management of Patients with Oral and
Esophageal Disorders 6. The school nurse is planning a health fair for a group of
1. A nurse is providing oral care to a patient who is fifth graders and dental health is one topic that the nurse
comatose. What action best addresses the patients risk plans to address. What would be most likely to increase
of tooth decay and plaque accumulation? the risk of tooth decay?
A) Irrigating the mouth using a syringe filled with a A) Organic fruit juice
bacteriocidal mouthwash B) Roasted nuts
B) Applying a water-soluble gel to the teeth and gums C) Red meat that is high in fat
8
D) Cheddar cheese nurses most recent assessment of the graft reveals that it
has a bluish color and that mottling is visible.
What is the nurses most appropriate action?
7. The nurses comprehensive assessment of a patient
includes inspection for signs of oral cancer. What A) Document the findings as being consistent with a viable
assessment finding is most characteristic of oral cancer in graft.
its early stages? B) Promptly report these indications of venous congestion.
A) Dull pain radiating to the ears and teeth C) Closely monitor the patient and reassess in 30 minutes.
B) Presence of a painless sore with raised edges D) Reposition the patient to promote peripheral
C) Areas of tenderness that make chewing difficult circulation.
D) Diffuse inflammation of the buccal mucosa
13. A nurse is assessing a patient who has just been
8. A medical nurse who is caring for a patient being admitted to the postsurgical unit following surgical
discharged home after a radical neck dissection has resection for the treatment of oropharyngeal cancer.
collaborated with the home health nurse to develop a What assessment should the nurse prioritize?
plan of care for this patient. What is a priority A) Assess ability to clear oral secretions.
psychosocial outcome for a patient who has had a radical B) Assess for signs of infection.
neck dissection? C) Assess for a patent airway.
A) Indicates acceptance of altered appearance and D) Assess for ability to communicate.
demonstrates positive self-image
B) Freely expresses needs and concerns related to 14. A patient has been diagnosed with achalasia based on
postoperative pain management his history and diagnostic imaging results. The nurse
C) Compensates effectively for alteration in ability to should identify what risk diagnosis when planning the
communicate related to dysarthria patients care?
D) Demonstrates effective stress management techniques A) Risk for Aspiration Related to Inhalation of Gastric
to promote muscle relaxation Contents
B) Risk for Imbalanced Nutrition: Less than Body
9. A patient has been diagnosed with an esophageal Requirements Related to Impaired Absorption
diverticulum after undergoing diagnostic imaging. C) Risk for Decreased Cardiac Output Related to Vasovagal
When taking the health history, the nurse should expect Response
the patient to describe what sign or symptom? D) Risk for Impaired Verbal Communication Related to
A) Burning pain on swallowing Oral Trauma
B) Regurgitation of undigested food
C) Symptoms mimicking a heart attack 15. A nurse is providing health promotion education to a
D) Chronic parotid abscesses patient diagnosed with an esophageal reflux disorder.
What practice should the nurse encourage the patient to
10. A nurse is caring for a patient who is acutely ill and implement?
has included vigilant oral care in the patients plan of A) Keep the head of the bed lowered.
care. Why are patients who are ill at increased risk for B) Drinka cup of hot tea before bedtime.
developing dental caries? C) Avoid carbonated drinks.
A) Hormonal changes brought on by the stress response D) Eat a low-protein diet.
cause an acidic oral environment
B) Systemic infections frequently migrate to the teeth 16. A staff educator is reviewing the causes of
C) Hydration that is received intravenously lacks fluoride gastroesophageal reflux disease (GERD) with new staff
D) Inadequate nutrition and decreased saliva production nurses. What area of the GI tract should the educator
can cause cavities identify as the cause of reduced pressure associated
with GERD?
11. A nurse who provides care in an ambulatory clinic A) Pyloric sphincter
integrates basic cancer screening into admission B) Lower esophageal sphincter
assessments. What patient most likely faces the highest C) Hypopharyngeal sphincter
immediate risk of oral cancer? D) Upper esophageal sphincter
A) A 65-year-old man with alcoholism who smokes
B) A 45-year-old woman who has type 1 diabetes and 17. A patient who has had a radical neck dissection is
who wears dentures being prepared for discharge. The discharge plan
C) A 32-year-old man who is obese and uses smokeless includes referral to an outpatient rehabilitation center for
tobacco physical therapy. What would the goals of
D) A 57-year-old man with GERD and dental caries physical therapy for this patient include?
A) Muscle training to relieve dysphagia
12. A nurse is caring for a patient who has undergone B) Relieving nerve paralysis in the cervical plexus
neck resection with a radial forearm free flap. The C) Promoting maximum shoulder function

9
D) Alleviating achalasia by decreasing esophageal A) Ineffective Tissue Perfusion
peristalsis B) Impaired Skin Integrity
C) Aspiration
D) Imbalanced Nutrition: Less Than Body Requirements
18. A nurse is addressing the prevention of esophageal 24. A patient has undergone rigid fixation for the
cancer in response to a question posed by a participant in correction of a mandibular fracture suffered in a fight.
a health promotion workshop. What action has the What area of care should the nurse prioritize when
greatest potential to prevent esophageal cancer? planning this patients discharge education?
A) Promotion of a nutrient-dense, low-fat diet A) Resumption of activities of daily living
B) Annual screening endoscopy for patients over 50 with B) Pain control
a family history of esophageal cancer C) Promotion of adequate nutrition
C) Early diagnosis and treatment of gastroesophageal
reflux disease 25. A radial graft is planned in the treatment of a patients
19. An emergency department nurse is admitting a 3- oropharyngeal cancer. In order to ensure that the surgery
year-old brought in after swallowing a piece from a will be successful, the care team must perform what
wooden puzzle. The nurse should anticipate the assessment prior to surgery?
administration of what medication in order to relax the A) Assessing function of cranial nerves V, VI, and IX
esophagus to facilitate removal of the foreign body? B) Assessing for a history of GERD
A) Haloperidol C) Assessing for signs or symptoms of atherosclerosis
B) Prostigmine D) Assessing the patency of the ulnar artery
C) Epinephrine
D) Glucagon 26. A nurse is caring for a patient who is postoperative day
1 following neck dissection surgery. The nurse is
20. A nurse in an oral surgery practice is working with a performing an assessment of the patient and notes the
patient scheduled for removal of an abscessed tooth. presence of high-pitched adventitious sounds over
When providing discharge education, the nurse should the patients trachea on auscultation. The patients oxygen
recommend which of the following actions? saturation is 90% by pulse oximetry with a respiratory rate
A) Rinse the mouth with alcohol before bedtime for the of 31 breaths per minute. What is the nurses most
next 7 days. appropriate action?
B) Use warm saline to rinse the mouth as needed. A) Encourage the patient to perform deep breathing and
C) Brush around the area with a firm toothbrush to coughing exercises hourly.
prevent infection. B) Reposition the patient into a prone or semi-Fowlers
D) Use a toothpick to dislodge any debris that gets lodged position and apply supplementary oxygen by nasal
in the socket. cannula.
C) Activate the emergency response system.
21. A patient has been diagnosed with a malignancy of D) Report this finding promptly to the physician and
the oral cavity and is undergoing oncologic treatment. remain with the patient.
The oncologic nurse is aware that the prognosis for
recovery from head and neck cancers is often poor 27. A nurse is caring for a patient who has just had a rigid
because of what characteristic of these malignancies? fixation of a mandibular fracture. When planning the
A) Radiation therapy often results in secondary brain discharge teaching for this patient, what would the nurse
tumors. be sure to include?
B) Surgical complications are exceedingly common. A) Increasing calcium intake to promote bone healing
C) Diagnosis rarely occurs until the cancer is endstage. B) Avoiding chewing food for the specified number of
D) Metastases are common and respond poorly to weeks after surgery
treatment. C) Techniques for managing parenteral nutrition in the
home setting
22. A patient has undergone surgery for oral cancer and D) Techniques for managing a gastrostomy
has just been extubated in postanesthetic recovery.
What nursing action best promotes comfort and 28. A community health nurse serves a diverse population.
facilitates spontaneous breathing for this patient? What individual would likely face the highest
A) Placing the patient in a left lateral position risk for parotitis?
B) Administering opioids as ordered A) A patient who is receiving intravenous antibiotic
C) Placing the patient in Fowlers position therapy in the home setting
D) Teaching the patient to use the patient-controlled B) A patient who has a chronic venous ulcer
analgesia (PCA) system C) An older adult whose medication regimen includes an
anticholinergic
23. A nurse is performing health education with a patient D) A patient with poorly controlled diabetes who receives
who has a history of frequent, serious dental caries. weekly wound care
When planning educational interventions, the nurse
should identify a risk for what nursing diagnosis?
10
When assessing the wound drainage over the first 24
postoperative hours the nurse would notify the physician
immediately for what?
29. A nurse is providing care for a patient whose neck A) Presence of small blood clots in the drainage
dissection surgery involved the use of a graft. When B) 60 mL of milky or cloudy drainage
assessing the graft, the nurse should prioritize data C) Spots of drainage on the dressings surrounding the
related to what nursing diagnosis? drain
A) Risk for Disuse Syndrome
B) Unilateral Neglect 36. A nurse is caring for a patient who is postoperative
C) Risk for Trauma from a neck dissection. What would be the most
D) Ineffective Tissue Perfusion appropriate nursing action to enhance the patients
appetite?
30. A patients neck dissection surgery resulted in damage A) Encourage the family to bring in the patients favored
to the patients superior laryngeal nerve. What area foods.
of assessment should the nurse consequently prioritize? B) Limit visitors at mealtimes so that the patient is not
A) The patients swallowing ability distracted.
B) The patients ability to speak C) Avoid offering food unless the patient initiates.
C) The patients management of secretions D) Provide thorough oral care immediately after the
D) The patients airway patency patient eats.

31. A patient who underwent surgery for esophageal 37. A patient with GERD has undergone diagnostic testing
cancer is admitted to the critical care unit following and it has been determined that increasing the pace of
postanesthetic recovery. Which of the following should gastric emptying may help alleviate symptoms. The nurse
be included in the patients immediate postoperative plan should anticipate that the patient may be prescribed what
of care? drug?
A) Teaching the patient to self-suction A) Metoclopramide (Reglan)
B) Performing chest physiotherapy to promote B) Omeprazole (Prilosec)
oxygenation C) Lansoprazole (Prevacid)
C) Positioning the patient to prevent gastric reflux D) Famotidine (Pepcid)
D) Providing a regular diet as tolerated
38. Results of a patient barium swallow suggest that the
32. A patient has received treatment for oral cancer. The patient has GERD. The nurse is planning health education
combination of medications and radiotherapy has to address the patients knowledge of this new diagnosis.
resulted in leukopenia. Which of the following is an Which of the following should the nurse encourage?
appropriate response to this change in health status? A) Eating several small meals daily rather than 3 larger
A) Ensure that none of the patients visitors has an meals
infection. B) Keeping the head of the bed slightly elevated
B) Arrange for a diet that is high in protein and low in fat. C) Drinking carbonated mineral water rather than soft
C) Administer colony stimulating factors (CSFs) as drinks
ordered. D) Avoiding food or fluid intake after 6:00 p.m.
D) Prepare to administer chemotherapeutics as ordered.
39. A nurse is caring for a patient in the late stages of
33. A nurse is caring for a patient who has had surgery for esophageal cancer. The nurse should plan to prevent
oral cancer. When addressing the patients longterm or address what characteristics of this stage of the
needs, the nurse should prioritize interventions and disease? Select all that apply.
referrals with what goal? A) Perforation into the mediastinum
A) Enhancement of verbal communication B) Development of an esophageal lesion
B) Enhancement of immune function C) Erosion into the great vessels
C) Maintenance of adequate social support D) Painful swallowing
D) Maintenance of fluid balance E) Obstruction of the esophagus

34. A patient with cancer of the tongue has had a radical 40. A patient seeking care because of recurrent heartburn
neck dissection. What nursing assessment would be and regurgitation is subsequently diagnosed with a
a priority for this patient? hiatal hernia. Which of the following should the nurse
A) Presence of acute pain and anxiety include in health education?
B) Tissue integrity and color of the operative site A) Drinking beverages after your meal, rather than with
C) Respiratory status and airway clearance your meal, may bring some relief.
D) Self-esteem and body image B) Its best to avoid dry foods, such as rice and chicken,
because theyre harder to swallow.
35. A patient returns to the unit after a neck dissection. C) Many patients obtain relief by taking over-the-counter
The surgeon placed a Jackson Pratt drain in the wound. antacids 30 minutes before eating.
11
D) Instead of eating three meals a day, try eating smaller 6. A nurse caring for a patient who has had bariatric
amounts more often. surgery is developing a teaching plan in anticipation of
the patients discharge. Which of the following is essential
Chapter 46: Management of Patients with Gastric and to include?
A) Drink a minimum of 12 ounces of fluid with each meal.
Duodenal Disorders
B) Eat several small meals daily spaced at equal intervals.
1. A nurse is caring for a patient who just has been
C) Choose foods that are high in simple carbohydrates.
diagnosed with a peptic ulcer. When teaching the
D) Sit upright when eating and for 30 minutes afterward.
patient about his new diagnosis, how should the nurse
best describe a peptic ulcer?
7. A nurse is completing a health history on a patient
A) Inflammation of the lining of the stomach
whose diagnosis is chronic gastritis. Which of the
B) Erosion of the lining of the stomach or intestine
data should the nurse consider most significantly related
C) Bleeding from the mucosa in the stomach
to the etiology of the patients health problem?
D) Viral invasion of the stomach wall
A) Consumes one or more protein drinks daily.
B) Takes over-the-counter antacids frequently throughout
2. A patient comes to the clinic complaining of pain in the
the day.
epigastric region. What assessment question
C) Smokes one pack of cigarettes daily.
during the health interview would most help the nurse
D) Reports a history of social drinking on a weekly basis.
determine if the patient has a peptic ulcer?
A) Does your pain resolve when you have something to
8. A nurse in the postanesthesia care unit admits a patient
eat?
following resection of a gastric tumor. Following
B) Do over-the-counter pain medications help your pain?
immediate recovery, the patient should be placed in which
C) Does your pain get worse if you get up and do some
position to facilitate patient comfort and gastric
exercise?
emptying?
D) Do you find that your pain is worse when you need to
A) Fowlers
have a bowel movement?
B) Supine
C) Left lateral
3. A patient with a diagnosis of peptic ulcer disease has
D) Left Sims
just been prescribed omeprazole (Prilosec). How should
the nurse best describe this medications therapeutic
9. A community health nurse is preparing for an initial
action?
home visit to a patient discharged following a total
A) This medication will reduce the amount of acid
gastrectomy for treatment of gastric cancer. What would
secreted in your stomach.
the nurse anticipate that the plan of care is most likely to
B) This medication will make the lining of your stomach
include?
more resistant to damage.
A) Enteral feeding via gastrostomy tube (G tube)
C) This medication will specifically address the pain that
B) Gastrointestinal decompression by nasogastric tube
accompanies peptic ulcer disease.
C) Periodic assessment for esophageal distension
D) Monthly administration of injections of vitamin B12
4. A nurse is admitting a patient diagnosed with late-stage
gastric cancer. The patients family is distraught and angry
10. A nurse is assessing a patient who has peptic ulcer
that she was not diagnosed earlier in the course of her
disease. The patient requests more information about
disease. What factor contributes to the fact that gastric
the typical causes of Helicobacter pylori infection. What
cancer is often detected at a later stage?
would it be appropriate for the nurse to instruct the
A) Gastric cancer does not cause signs or symptoms until
patient?
metastasis has occurred.
A) Most affected patients acquired the infection during
B) Adherence to screening recommendations for gastric
international travel.
cancer is exceptionally low.
B) Infection typically occurs due to ingestion of
C) Early symptoms of gastric cancer are usually attributed
contaminated food and water.
to constipation.
C) Many people possess genetic factors causing a
D) The early symptoms of gastric cancer are usually not
predisposition to H. pylori infection.
alarming or highly unusual.
D) The H. pylori microorganism is endemic in warm, moist
climates.
5. A nurse is preparing to discharge a patient after
Ans:
recovery from gastric surgery. What is an appropriate
discharge outcome for this patient?
11. A patient who experienced an upper GI bleed due to
A) The patients bowel movements maintain a loose
gastritis has had the bleeding controlled and the patients
consistency.
condition is now stable. For the next several hours, the
B) The patient is able to tolerate three large meals a day.
nurse caring for this patient should assess for what signs
C) The patient maintains or gains weight.
and symptoms of recurrence?
D) The patient consumes a diet high in calcium.
A) Tachycardia, hypotension, and tachypnea
B) Tarry, foul-smelling stools
12
C) Diaphoresis and sudden onset of abdominal pain should be informed that he may experience which of the
D) Sudden thirst, unrelieved by oral fluid administration following adverse effects associated with this procedure?
A) Persistent feelings of hunger and thirst
B) Constipation or bowel incontinence
12. A patient presents to the walk-in clinic complaining of C) Diarrhea and feelings of fullness
vomiting and burning in her mid-epigastria. The nurse 18. A patient has experienced symptoms of dumping
knows that in the process of confirming peptic ulcer syndrome following bariatric surgery. To what
disease, the physician is likely to order a diagnostic test to physiologic phenomenon does the nurse attribute this
detect the presence of what? syndrome?
A) Infection with Helicobacter pylori A) Irritation of the phrenic nerve due to diaphragmatic
B) Excessive stomach acid secretion pressure
C) An incompetent pyloric sphincter B) Chronic malabsorption of iron and vitamins A and C
D) A metabolic acidbase imbalance C) Reflux of bile into the distal esophagus
D) A sudden release of peptides
13. A patient with a peptic ulcer disease has had
metronidazole (Flagyl) added to his current medication 19. A patient comes to the bariatric clinic to obtain
regimen. What health education related to this information about bariatric surgery. The nurse assesses
medication should the nurse provide? the obese patient knowing that in addition to meeting the
A) Take the medication on an empty stomach. criterion of morbid obesity, a candidate for bariatric
B) Take up to one extra dose per day if stomach pain surgery must also demonstrate what?
persists. A) Knowledge of the causes of obesity and its associated
C) Take at bedtime to mitigate the effects of drowsiness. risks
D) Avoid drinking alcohol while taking the drug. B) Adequate understanding of required lifestyle changes
C) Positive body image and high self-esteem
14. A patient was treated in the emergency department D) Insight into why past weight loss efforts failed
and critical care unit after ingesting bleach. What
possible complication of the resulting gastritis should the 20. A nurse is providing patient education for a patient
nurse recognize? with peptic ulcer disease secondary to chronic
A) Esophageal or pyloric obstruction related to scarring nonsteroidal anti-inflammatory drug (NSAID) use. The
B) Uncontrolled proliferation of H. pylori patient has recently been prescribed misoprostol
C) Gastric hyperacidity related to excessive gastrin (Cytotec). What would the nurse be most accurate in
secretion informing the patient about the drug?
D) Chronic referred pain in the lower abdomen A) It reduces the stomachs volume of hydrochloric acid
B) It increases the speed of gastric emptying
15. A patient who underwent gastric banding 3 days ago C) It protects the stomachs lining
is having her diet progressed on a daily basis. Following D) It increases lower esophageal sphincter pressure
her latest meal, the patient complains of dizziness and
palpitations. Inspection reveals that the patient is 21. A nurse is providing anticipator guidance to a patient
diaphoretic. What is the nurses best action? who is preparing for bariatric surgery. The nurse learns
A) Insert a nasogastric tube promptly. that the patient is anxious about numerous aspects of the
B) Reposition the patient supine. surgery. What intervention is most appropriate to alleviate
C) Monitor the patient closely for further signs of the patients anxiety?
dumping syndrome. A) Emphasize the fact that bariatric surgery has a low risk
D) Assess the patient for signs and symptoms of of complications.
aspiration. B) Encourage the patient to focus on the benefits of the
surgery.
16. A patient is one month postoperative following C) Facilitate the patients contact with a support group.
restrictive bariatric surgery. The patient tells the clinic D) Obtain an order for a PRN benzodiazepine.
nurse that he has been having trouble swallowing for the
past few days. What recommendation should the nurse 22. A patient has just been diagnosed with acute gastritis
make? after presenting in distress to the emergency department
A) Eating more slowly and chewing food more thoroughly with abdominal symptoms. What would be the nursing
B) Taking an OTC antacid or drinking a glass of milk prior care most needed by the patient at this time?
to each meal A) Teaching the patient about necessary nutritional
C) Chewing gum to cause relaxation of the lower modification
esophageal sphincter B) Helping the patient weigh treatment options
D) Drinking at least 12 ounces of liquid with each meal C) Teaching the patient about the etiology of gastritis
D) Providing the patient with physical and emotional
17. A patient is receiving education about his upcoming support
Billroth I procedure (gastroduodenostomy). This patient

13
28. Diagnostic imaging and physical assessment have
revealed that a patient with peptic ulcer disease has
suffered a perforated ulcer. The nurse recognizes that
emergency interventions must be performed as
soon as possible in order to prevent the development of
23. A nurse is providing care for a patient who is what complication?
postoperative day 2 following gastric surgery. The nurses A) Peritonitis
assessment should be planned in light of the possibility of B) Gastritis
what potential complications? Select all that apply. C) Gastroesophageal reflux
A) Malignant hyperthermia D) Acute pancreatitis
B) Atelectasis
C) Pneumonia 29. A nurse is performing the admission assessment of a
D) Metabolic imbalances patient whose high body mass index (BMI) corresponds to
E) Chronic gastritis class III obesity. In order to ensure empathic and patient-
centered care, the nurse should
24. A patient is undergoing diagnostic testing for a tumor do which of the following?
of the small intestine. What are the most likely A) Examine ones own attitudes towards obesity in general
symptoms that prompted the patient to first seek care? and the patient in particular.
A) Hematemesis and persistent sensation of fullness B) Dialogue with the patient about the lifestyle and
B) Abdominal bloating and recurrent constipation psychosocial factors that resulted in obesity.
C) Intermittent pain and bloody stool C) Describe ones own struggles with weight gain and
D) Unexplained bowel incontinence and fatty stools weight loss to the patient.
D) Elicit the patients short-term and long-term goals for
25. A patient is recovering in the hospital following weight loss.
gastrectomy. The nurse notes that the patient has
become increasingly difficult to engage and has had 30. A patient has been prescribed orlistat (Xenical) for the
several angry outbursts at various staff members in treatment of obesity. When providing relevant health
recent days. The nurses attempts at therapeutic dialogue education for this patient, the nurse should ensure the
have been rebuffed. What is the nurses most appropriate patient is aware of what potential adverse effect of
action? treatment?
A) Ask the patients primary care provider to liaise A) Bowel incontinence
between the nurse and the patient. B) Flatus with oily discharge
B) Delegate care of the patient to a colleague. C) Abdominal pain
C) Limit contact with the patient in order to provide D) Heat intolerance
privacy.
D) Make appropriate referrals to services that provide 31. A patient who is obese has been unable to lose weight
psychosocial support. successfully using lifestyle modifications and has
mentioned the possibility of using weight-loss
26. A patient has been admitted to the hospital after medications. What should the nurse teach the patient
diagnostic imaging revealed the presence of a gastric about pharmacologic interventions for the treatment of
outlet obstruction (GOO). What is the nurses priority obesity?
intervention? A) Weight loss drugs have many side effects, and most
A) Administration of antiemetics doctors think theyll all be off the market in a few years.
B) Insertion of an NG tube for decompression B) There used to be a lot of hope that medications would
C) Infusion of hypotonic IV solution help people lose weight, but its been shown to be mostly a
D) Administration of proton pump inhibitors as ordered placebo effect.
C) Medications can be helpful, but few people achieve and
27. A patient with a history of peptic ulcer disease has maintain their desired weight loss with medications alone.
presented to the emergency department (ED) in
distress. What assessment finding would lead the ED 32. A patient has been diagnosed with peptic ulcer disease
nurse to suspect that the patient has a perforated and the nurse is reviewing his prescribed medication
ulcer? regimen with him. What is currently the most commonly
A) The patient has abdominal bloating that developed used drug regimen for peptic ulcers?
rapidly. A) Bismuth salts, antivirals, and histamine-2 (H2)
B) The patient has a rigid, boardlike abdomen that is antagonists
tender. B) H2 antagonists, antibiotics, and bicarbonate salts
C) The patient is experiencing intense lower right C) Bicarbonate salts, antibiotics, and ZES
quadrant pain. D) Antibiotics, proton pump inhibitors, and bismuth salts
D) The patient is experiencing dizziness and confusion
with no apparent hemodynamic changes.

14
38. A patient with gastritis required hospital treatment for
an exacerbation of symptoms and receives a
subsequent diagnosis of pernicious anemia due to
malabsorption. When planning the patients continuing
33. A patient who is obese is exploring bariatric surgery care in the home setting, what assessment question is
options and presented to a bariatric clinic for most relevant?
preliminary investigation. The nurse interviews the A) Does anyone in your family have experience at giving
patient, analyzing and documenting the data. Which injections?
of the following nursing diagnoses may be a B) Are you going to be anywhere with strong sunlight in
contraindication for bariatric surgery? the next few months?
A) Disturbed Body Image Related to Obesity C) Are you aware of your blood type?
B) Deficient Knowledge Related to Risks and Expectations D) Do any of your family members have training in first
of Surgery aid?
C) Anxiety Related to Surgery
D) Chronic Low Self-Esteem Related to Obesity 39. A nurse is presenting a class at a bariatric clinic about
the different types of surgical procedures offered by the
34. A patient has recently received a diagnosis of gastric clinic. When describing the implications of different types
cancer; the nurse is aware of the importance of of surgeries, the nurse should address which of the
assessing the patients level of anxiety. Which of the following topics? Select all that apply.
following actions is most likely to accomplish this? A) Specific lifestyle changes associated with each
A) The nurse gauges the patients response to procedure
hypothetical outcomes. B) Implications of each procedure for eating habits
B) The patient is encouraged to express fears openly. C) Effects of different surgeries on bowel function
C) The nurse provides detailed and accurate information D) Effects of various bariatric surgeries on fertility
about the disease. E) Effects of different surgeries on safety of future
D) The nurse closely observes the patients body language. immunizations

35. A patient has received a diagnosis of gastric cancer 40. A patient has come to the clinic complaining of pain
and is awaiting a surgical date. During the just above her umbilicus. When assessing the patient, the
preoperative period, the patient should adopt what nurse notes Sister Mary Josephs nodules. The nurse
dietary guidelines? should refer the patient to the primary care provider to be
A) Eat small, frequent meals with high calorie and vitamin assessed for what health problem?
content. A) A GI malignancy
B) Eat frequent meals with an equal balance of fat, B) Dumping syndrome
carbohydrates, and protein. C) Peptic ulcer disease
C) Eat frequent, low-fat meals with high protein content. D) Esophageal/gastric obstruction
D) Try to maintain the pre-diagnosis pattern of eating.
Chapter 47: Management of Patients With Intestinal
36. A nurse is caring for a patient who has a diagnosis of and Rectal Disorders
GI bleed. During shift assessment, the nurse finds the 1. A nurse is working with a patient who has chronic
patient to betachycardic and hypotensive, and the patient constipation. What should be included in patient
has an episode of hematemesis while the nurse is in the teaching to promote normal bowel function?
room. In addition to monitoring the patients vital signs A) Use glycerin suppositories on a regular basis.
and level of conscious, what would be a priority nursing B) Limit physical activity in order to promote bowel
action for this patient? peristalsis.
A) Place the patient in a prone position. C) Consume high-residue, high-fiber foods.
B) Provide the patient with ice water to slow any GI
bleeding. 2. A nurse is preparing to provide care for a patient whose
C) Prepare for the insertion of an NG tube. exacerbation of ulcerative colitis has required hospital
D) Notify the physician. admission. During an exacerbation of this health problem,
the nurse would anticipate that the patients stools will
37. A nurse is caring for a patient hospitalized with an have what characteristics?
exacerbation of chronic gastritis. What health A) Watery with blood and mucus
promotion topic should the nurse emphasize? B) Hard and black or tarry
A) Strategies for maintaining an alkaline gastric C) Dry and streaked with blood
environment D) Loose with visible fatty streaks
B) Safe technique for self-suctioning
C) Techniques for positioning correctly to promote gastric 3. A patient has had an ileostomy created for the
healing treatment of irritable bowel disease and the patient is
D) Strategies for avoiding irritating foods and beverages now preparing for discharge. What should the patient be
taught about changing this device in the home setting?
15
A) Apply antibiotic ointment as ordered after cleaning the A) Adherence to a high-fiber diet will help the polyps
stoma. resolve.
B) Apply a skin barrier to the peristomal skin prior to B) The patient should be assured that these are a normal,
applying the pouch. age-related physiologic change.
C) Dispose of the clamp with each bag change. C) The patients polyps constitute a risk factor for cancer.
4. A patient admitted with acute diverticulitis has 10. A nursing instructor is discussing hemorrhoids with the
experienced a sudden increase in temperature and nursing class. Which patients would the nursing
complains of a sudden onset of exquisite abdominal instructor identify as most likely to develop hemorrhoids?
tenderness. The nurses rapid assessment reveals that A) A 45-year-old teacher who stands for 6 hours per day
the patients abdomen is uncharacteristically rigid on B) A pregnant woman at 28 weeks gestation
palpation. What is the nurses best response? C) A 37-year-old construction worker who does heavy
A) Administer a Fleet enema as ordered and remain with lifting
the patient. D) A 60-year-old professional who is under stress
B) Contact the primary care provider promptly and report
these signs of perforation. 11. A nurse is planning discharge teaching for a 21-year-
C) Position the patient supine and insert an NG tube. old patient with a new diagnosis of ulcerative colitis.
D) Page the primary care provider and report that the When planning family assessment, the nurse should
patient may be obstructed. recognize that which of the following factors will likely
have the greatest impact on the patients coping after
5. A 35-year-old male patient presents at the emergency discharge?
department with symptoms of a small bowel obstruction. A) The familys ability to take care of the patients special
In collaboration with the primary care provider, what diet needs
intervention should the nurse prioritize? B) The familys ability to monitor the patients changing
A) Insertion of a nasogastric tube health status
B) Insertion of a central venous catheter C) The familys ability to provide emotional support
C) Administration of a mineral oil enema D) The familys ability to manage the patients medication
D) Administration of a glycerin suppository and an oral regimen
laxative
12. An older adult who resides in an assisted living facility
6. A patient admitted with inflammatory bowel disease has sought care from the nurse because of recurrent
asks the nurse for help with menu selections. What episodes of constipation. Which of the following actions
menu selection is most likely the best choice for this should the nurse first perform?
patient? A) Encourage the patient to take stool softener daily.
A) Spinach B) Assess the patients food and fluid intake.
B) Tofu C) Assess the patients surgical history.
C) Multigrain bagel D) Encourage the patient to take fiber supplements.
D) Blueberries
13. A 16-year-old presents at the emergency department
7. A patient is admitted to the medical unit with a complaining of right lower quadrant pain and is
diagnosis of intestinal obstruction. When planning this subsequently diagnosed with appendicitis. When planning
patients care, which of the following nursing diagnoses this patients nursing care, the nurse should prioritize what
should the nurse prioritize? nursing diagnosis?
A) Ineffective Tissue Perfusion Related to Bowel Ischemia A) Imbalanced Nutrition: Less Than Body Requirements
B) Imbalanced Nutrition: Less Than Body Requirements Related to Decreased Oral Intake
Related to Impaired Absorption B) Risk for Infection Related to Possible Rupture of
C) Anxiety Related to Bowel Obstruction and Subsequent Appendix
Hospitalization C) Constipation Related to Decreased Bowel Motility and
D) Impaired Skin Integrity Related to Bowel Obstruction Decreased Fluid Intake
D) Chronic Pain Related to Appendicitis
8. A nurse is presenting an educational event to a local
community group. When speaking about colorectal 14. A nurse is talking with a patient who is scheduled to
cancer, what risk factor should the nurse cite? have a hemicolectomy with the creation of a colostomy.
A) High levels of alcohol consumption The patient admits to being anxious, and has many
B) History of bowel obstruction questions concerning the surgery, the care of a stoma, and
C) History of diverticulitis necessary lifestyle changes. Which of the following nursing
D) Longstanding psychosocial stress actions is most appropriate?
A) Reassure the patient that the procedure is relatively
9. A patients screening colonoscopy revealed the low risk and that patients are usually successful in
presence of numerous polyps in the large bowel. What adjusting to an ostomy.
principle should guide the subsequent treatment of this B) Provide the patient with educational materials that
patients health problem? match the patients learning style.
16
C) Encourage the patient to write down these concerns B) Report signs and symptoms of obstruction to the
and questions to bring forward to the surgeon. physician.
D) Maintain an open dialogue with the patient and C) Encourage the patient to mobilize in order to enhance
facilitate a referral to the wound-ostomy- motility.
continence (WOC) nurse.
15. A nurse is caring for a patient with constipation whose 21. A nurse is working with a patient who is learning to
primary care provider has recommended senna (Senokot) care for a continent ileostomy (Kock pouch). Following the
for the management of this condition. The nurse should initial period of healing, the nurse is teaching the patient
provide which of the following education points? how to independently empty the ileostomy. The nurse
A) Limit your fluid intake temporarily so you dont get should teach the patient to do which of the following
diarrhea. actions?
B) Avoid taking the drug on a long-term basis. A) Aim to eventually empty the pouch every 90 minutes.
C) Make sure to take a multivitamin with each dose. B) Avoid emptying the pouch until it is visibly full.
D) Take this on an empty stomach to ensure maximum C) Insert the catheter approximately 5 cm into the pouch.
effect. D) Aspirate the contents of the pouch using a 60 mL piston
syringe.
16. The nurse is caring for a patient who is undergoing
diagnostic testing for suspected malabsorption. When 22. A nurse is providing care for a patient who has a
taking this patients health history and performing the diagnosis of irritable bowel syndrome (IBS). When
physical assessment, the nurse should recognize what planning this patients care, the nurse should collaborate
finding as most consistent with this diagnosis? with the patient and prioritize what goal?
A) Recurrent constipation coupled with weight loss A) Patient will accurately identify foods that trigger
B) Foul-smelling diarrhea that contains fat symptoms.
C) Fever accompanied by a rigid, tender abdomen B) Patient will demonstrate appropriate care of his
D) Bloody bowel movements accompanied by fecal ileostomy.
incontinence C) Patient will demonstrate appropriate use of standard
infection control precautions.
17. A nurse is caring for a patient admitted with D) Patient will adhere to recommended guidelines for
symptoms of an anorectal infection; cultures indicate that mobility and activity.
the patient has a viral infection. The nurse should
anticipate the administration of what drug? 23. A patient has been experiencing disconcerting GI
A) Acyclovir (Zovirax) symptoms that have been worsening in severity.
B) Doxycycline (Vibramycin) Following medical assessment, the patient has been
C) Penicillin (penicillin diagnosed with lactose intolerance. The nurse should
D) Metronidazole (Flagyl) recognize an increased need for what form of health
promotion?
18. A nurse caring for a patient with colorectal cancer is A) Annual screening colonoscopies
preparing the patient for upcoming surgery. The nurse B) Adherence to recommended immunization schedules
administers cephalexin (Keflex) to the patient and C) Regular blood pressure monitoring
explains what rationale? D) Frequent screening for osteoporosis
A) To treat any undiagnosed infections
B) To reduce intestinal bacteria levels 24. An older adult has a diagnosis of Alzheimers disease
C) To reduce bowel motility and has recently been experiencing fecal incontinence.
D) To reduce abdominal distention postoperatively However, the nurse has observed no recent change in the
character of the patients stools. What is the nurses most
19. A nurse is teaching a group of adults about screening appropriate intervention?
and prevention of colorectal cancer. The nurse should A) Keep a food diary to determine the foods that
describe which of the following as the most common sign exacerbate the patients symptoms.
of possible colon cancer? B) Provide the patient with a bland, low-residue diet.
A) Development of new hemorrhoids C) Toilet the patient on a frequent, scheduled basis.
B) Abdominal bloating and flank pain D) Liaise with the primary care provider to obtain an order
C) Unexplained weight gain for loperamide.
D) Change in bowel habits
25. An adult patient has been diagnosed with diverticular
20. A nurse caring for a patient with a newly created disease after ongoing challenges with constipation.
ileostomy assesses the patient and notes that the patient The patient will be treated on an outpatient basis. What
has had not ostomy output for the past 12 hours. The components of treatment should the nurse
patient also complains of worsening nausea. What anticipate? Select all that apply.
is the nurses priority action? A) Anticholinergic medications
A) Facilitate a referral to the wound-ostomy-continence B) Increased fiber intake
(WOC) nurse. C) Enemas on alternating days
17
D) Reduced fat intake
E) Fluid reduction

31. A patients colorectal cancer has necessitated a


26. A patients health history is suggestive of inflammatory hemicolectomy with the creation of a colostomy. In the 4
bowel disease. Which of the following would suggest days since the surgery, the patient has been unwilling to
Crohns disease, rather that ulcerative colitis, as the cause look at the ostomy or participate in any aspects of ostomy
of the patients signs and symptoms? care. What is the nurses most appropriate response to this
A) A pattern of distinct exacerbations and remissions observation?
B) Severe diarrhea A) Ensure that the patient knows that he or she will be
C) An absence of blood in stool responsible for care after discharge.
D) Involvement of the rectal mucosa B) Reassure the patient that many people are fearful after
the creation of an ostomy.
27. During a patients scheduled home visit, an older adult C) Acknowledge the patients reluctance and initiate
patient has stated to the community health nurse that discussion of the factors underlying it.
she has been experiencing hemorrhoids of increasing D) Arrange for the patient to be seen by a social worker or
severity in recent months. The nurse should spiritual advisor.
recommend which of the following?
A) Regular application of an OTC antibiotic ointment 32. A nurse is caring for an older adult who has been
B) Increased fluid and fiber intake experiencing severeClostridium difficile-related diarrhea.
C) Daily use of OTC glycerin suppositories When reviewing the patients most recent laboratory tests,
D) Use of an NSAID to reduce inflammation the nurse should prioritize which of
the following?
28. A nurse is providing care for a patient whose recent A) White blood cell level
colostomy has contributed to a nursing diagnosis of B) Creatinine level
Disturbed Body Image Related to Colostomy. What C) Hemoglobin level
intervention best addresses this diagnosis? D) Potassium level
A) Encourage the patient to conduct online research into
colostomies. 33. A nurse is assessing a patients stoma on postoperative
B) Engage the patient in the care of the ostomy to the day 3. The nurse notes that the stoma has a shiny
extent that the patient is willing. appearance and a bright red color. How should the nurse
C) Emphasize the fact that the colostomy was needed to best respond to this assessment finding?
alleviate a much more serious health A) Irrigate the ostomy to clear a possible obstruction.
problem. B) Contact the primary care provider to report this finding.
D) Emphasize the fact that the colostomy is temporary C) Document that the stoma appears healthy and well
measure and is not permanent. perfused.
D) Document a nursing diagnosis of Impaired Skin
29. A nurse is caring for a patient who has been admitted Integrity.
to the hospital with diverticulitis. Which of the following
would be appropriate nursing diagnoses for this patient? 34. A patient has been diagnosed with a small bowel
Select all that apply. obstruction and has been admitted to the medical unit.
A) Acute Pain Related to Increased Peristalsis and GI The nurses care should prioritize which of the following
Inflammation outcomes?
B) Activity Intolerance Related to Generalized Weakness A) Preventing infection
C) Bowel Incontinence Related to Increased Intestinal B) Maintaining skin and tissue integrity
Peristalsis C) Preventing nausea and vomiting
D) Deficient Fluid Volume Related to Anorexia, Nausea, D) Maintaining fluid and electrolyte balance
and Diarrhea
E) Impaired Urinary Elimination Related to GI Pressure on 35. A patients large bowel obstruction has failed to resolve
the Bladder spontaneously and the patients worsening condition has
warranted admission to the medical unit. Which of the
30. The nurse is providing care for a patient whose following aspects of nursing care is most appropriate for
inflammatory bowel disease has necessitated hospital this patient?
treatment. Which of the following would most likely be A) Administering bowel stimulants as ordered
included in the patients medication regimen? B) Administering bulk-forming laxatives as ordered
A) Anticholinergic medications 30 minutes before a meal C) Performing deep palpation as ordered to promote
B) Antiemetics on a PRN basis peristalsis
C) Vitamin B12 injections to prevent pernicious anemia D) Preparing the patient for surgical bowel resection
D) Beta adrenergic blockers to reduce bowel motility

18
36. A patient has been experiencing occasional episodes
of constipation and has been unable to achieve
consistent relief by increasing physical activity and
improving his diet. What pharmacologic intervention
should the nurse recommend to the patient for ongoing
use?
A) Mineral oil enemas
B) Bisacodyl (Dulcolax)
C) Senna (Senokot)
D) Psyllium hydrophilic mucilloid (Metamucil)

37. A patient with a diagnosis of colon cancer is 2 days


postoperative following bowel resection and
anastomosis. The nurse has planned the patients care in
the knowledge of potential complications. What
assessment should the nurse prioritize?
A) Close monitoring of temperature
B) Frequent abdominal auscultation
C) Assessment of hemoglobin, hematocrit, and red blood
cell levels
D) Palpation of peripheral pulses and leg girth

38. A teenage patient with a pilonidal cyst has been


brought for care by her mother. The nurse who is
contributing to the patients care knows that treatment
will be chosen based on what risk?
A) Risk for infection
B) Risk for bowel incontinence
C) Risk for constipation
D) Risk for impaired tissue perfusion

39. A nurse at an outpatient surgery center is caring for a


patient who had a hemorrhoidectomy. What discharge
education topics should the nurse address with this
patient?
A) The appropriate use of antibiotics to prevent
postoperative infection
B) The correct procedure for taking a sitz bath
C) The need to eat a low-residue, low-fat diet for the next
2 weeks
D) The correct technique for keeping the perianal region
clean without the use of water

40. Which of the following is the most plausible nursing


diagnosis for a patient whose treatment for colon
cancer has necessitated a colonostomy?
A) Risk for Unstable Blood Glucose Due to Changes in
Digestion and Absorption
B) Unilateral Neglect Related to Decreased Physical
Mobility
C) Risk for Excess Fluid Volume Related to Dietary
Changes and Changes In Absorption
D) Ineffective Sexuality Patterns Related to Changes in
Self-Concept

19
b. Semi-Fowler position with the left leg bent to minimize
pressure on the abdomen
c. Supine position with the knees flexed to relax the
CHAPTER 43 abdominal muscles d. Reverse Trendelenburg position to
facilitate the natural propulsion of intestinal contents
1. The nurse is performing an initial assessment of a
patient reporting increased stomach acid related to 8. The nurse auscultates a patient's abdomen to assess
stress. The nurse knows that the healthcare provider will bowel sounds, documenting five to six sounds heard in
want to consider the influence of what neuroregulator? less than 30 seconds. How does the nurse document the
a. Gastrin character of the bowel sounds?
b. Cholecystokinin a. Normal
c. Norepinephrine b. Hypoactive
d. Secretin c. Hyperactive
d. Borborygmi
2. The nurse is performing an assessment of a patient.
During the assessment the patient informs the nurse of 9. The nurse is providing instructions to a patient
some recent "stomach trouble. " What does the nurse scheduled for a gastroscopy. What should the nurse be
recognize is the most common symptom of patients with sure to include in the instructions? (Select all that apply.)
GI dysfunction? a. The patient must fast for 8 hours before the
a. Diffuse pain examination
b. Dyspepsia b. The throat will be sprayed with a local anesthetic
c. Constipation c. After gastroscopy, the patient cannot eat or drink until
d. Abdominal bloating the gag reflex returns (1 to 2 hours)
d. The healthcare provider will be able to determine if
3. The nurse is investigating a patient's report of pain in there is a presence of bowel disease e. The patient must
the duodenal area. Where should the nurse perform the have bowel cleansing prior to the procedure
assessment?
a. Epigastric area and consider possible radiation of pain 10. A patient is scheduled for a fiberoptic colo-noscopy.
to the right subscapular region What does the nurse tell the patient fiberoptic
b. Hypogastrium in the right or left lower quadrant colonoscopy is most frequently used to diagnose?
c. Left lower quadrant a. Bowel disease of unknown origin
d. Periumbilical area, followed by the right lower b. Cancer screening
quadrant c. Inflammatory bowel disease
d. Occult bleeding
4. A patient reports abdominal pain associated with
indigestion. What is characteristic of this type of pain? 11. A patient is being prepared for esophageal
a. Described as crampy or burning manometry. The nurse should inform the patient to
b. In the left lower quadrant withhold what medication for 48 hours prior to the
c. Less severe after an intake of fatty foods procedure?
d. Relieved by the intake of coarse vegetables, which a. Amiodarone
stimulate peristalsis b. Calan
C. Aspirin
5. The nurse is collecting a stool specimen from a patient. d. Metoprolol
What characteristic of the stool indicates to the nurse
that the patient may have an upper GI bleed? 12. The nurse is assisting the healthcare provider with a
a. Clay colored gastric acid stimulation test for a patient. What
b. Greasy and foamy medication should the nurse prepare to administer
c. Tarry and black subcutaneously to stimulate gastric secretions?
d. Threaded with mucus a. Pentagastrin
b. Atropine
6. The nurse is performing an abdominal assessment for a c. Glycopyrrolate
patient in the hospital with reports of abdominal pain. d. Acetylcysteine
What part of the assessment should the nurse perform
first? a. Percussion b. Palpation c. Auscultation a. 13. The nurse is assisting the healthcare provider with a
Inspection colonoscopy for a patient with rectal bleeding. The nurse
is asked to administer glucagon during the procedure.
7. The nurse has been directed to position a patient for an Why is the nurse asked to administer this medication
examination of the abdomen. What position should the during the procedure?
nurse place the patient in for the examination? a. The patient is probably hypoglycemic and requires the
a. Prone position with pillows positioned to alleviate glucagon
pressure on the abdomen
20
b. To relieve anxiety during the procedure for moderate confirm placement, What does the nurse anticipate the pH
sedation will be if placement is in the lungs?
c. To reduce air accumulation in the colon а. 1
d. To relax colonic musculature and reduce spasm b. 2
14. A patient is in the outpatient recovery area after C. 4
having a colonoscopy and informs the nurse of abdominal d. 6
cramping. What is the best response by the nurse?
a. "We may need to go back in and see what - is wrong. 5. The nurse is managing a gastric Salem) sump tube for a
You shouldn't have discomfort" patient who has an intestinal obstruction and will be going
b. "I will call the physician and let him know. He may have to surgery. What interventions should be the nurse
put too much air in your colon" perform to make sure the tube is functioning properly?
c. "I will call the physician and see if I can give you pain a. Maintain intermittent or continuous suction at a rate
medication. Sometimes the pain can be caused by having greater than 120 mm Hg
a biopsy" b. Keep the vent lumen above the patient's waist to
d. "The cramping is caused by the air insuf-flated in the prevent gastric content reflux
colon during the procedure" c. Irrigate only through the vent lumen
d. Tape the tube to the head of the bed to avoid
15. During a colonoscopy with moderate seda-tion, the dislodgement
patient groans with obvious discomfort and begins
bleeding from the rectum. The patient is diaphoretic and 6. The nurse is inserting a nasoenteric tube for a patient
has an increase in abdominal girth from distention. What with a paralytic ileus. How long does the nurse anticipate
complication of this procedure is the nurse aware may be the tube will be required? (Select all that apply.) a.) Until
occurring? bowel sound is present (b. Until flatus is passed c. Until
a. Infection peristalsis is resumed d. Until the patient stops vomiting e.
B. Bowel perforation Until the tube comes out on its own
c. Colonic polyp
d. Rectal fissure 7. The nurse assesses a patient who recently had a
nasoenteric intubation. Symptoms of oligu-ria, lethargy,
CHAPTER 44 and tachycardia in the patient would indicate to the nurse
what common complication? a. A cardiac dysrhythmia b,
1. The healthcare provider prescribes a naso-enteric Fluid volume deficit c. Mucous membrane irritation d.
feeding tube with a tungsten-weighted tip. The nurse Pulmonary complications
knows to obtain what kind of tube for insertion?
a. A Dobbhoff or EnteraFio tube 8. The nurse checks residual content before each
b. A Levin tube intermittent tube feeding. When should the patient be
c. A Salem Sump tube reassessed?
d. A Sengstaken-Blakemore tube a. When the residual is about 50 mL
b. When the residual is between 50 and 80 mL
2. A nurse prepares a patient for insertion of a c. When the residual is about 100 mL
nasoenteric tube. What position should the nurse place d. When the residual is greater than 200 ml
the patient in to facilitate insertion?
(a) In high-Fowler position 9. The nurse is caring for a patient who has dumping
b. Flat in bed syndrome from high-carbohydrate foods being
c. On their right side administered over a period of less than 20 minutes. What
d. In' semi-Fowler position with their head turned to the is a nursing measure to prevent or minimize the dumping
left syndrome?
a. Administer the feeding at a warm temperature to
3. The nurse is inserting a Levin tube for a patient for decrease peristalsis
gastric decompression. The tube should be inserted to 6 b. Administer the feeding by bolus to prevent continuous
to 10 cm beyond what length? intestinal distention
a. A length of 50 cm (20 in) c. Administer the feeding with about 100 mL of fluid to
b. b. A point that equals the distance from the nose to the dilute the high-carbohydrate concentration
xiphoid process d. Administer the feeding with the patient in semi-Fowler
c. The distance measured from the tip of the nose to the position to decrease transit time influenced by gravity
earlobe and from the earlobe to the xiphoid process
d. The distance determined by measuring. from the 10. The nurse is caring for a comatose patient and
tragus of the ear to the xiphoid process administering gastrostomy feedings. What does the nurse
understand is the reason that gastrostomy feedings are
4. The nurse inserts a nasogastric tube into the right nares preferred to nasogastric feedings in the comatose patient?
of a patient. When testing the tube aspirate for pH to a. Gastroesophageal sphincter is intact, lessening the
possibility of regurgitation
21
b. Digestive process occurs more rapidly because the What does the nurse suspect that these findings are
feedings do not have to pass through the esophagus characteristic of?
c. Feedings can be administered with the patient in the a. Actinic cheilitis
recumbent position b. Human papillomavirus lesion
11. A patient has had a gastrostomy tube inserted. What c. Frey syndrome
does the nurse anticipate the initial fluid nourishment will d. Sialadenitis
be after the insertion of the gastrostomy tube?
a. Distilled water 2. A patient is experiencing painful, inflamed, and swollen
b. 10% glucose and tap water gums, and when brushing the teeth, the gums bleed. What
c. Milk common disease of the oral tissue does the nurse
d. High-calorie liquids understand these symptoms indicate?
a. Candidiasis
12. The nurse is caring for a patient who has a b. Gingivitis
gastrostomy tube feeding. Upon initiating her care, the c. Herpes simplex
nurse aspirates the gastrostomy tube for gastric residual d. Cancer of the oral mucosa
volume (GRV) and obtains 200 mL, of gastric contents.
What is the priority action by the nurse? 3. The nurse is caring for a patient after drainage of a
a. Discontinue the infusion dentoalveolar or periapical abscess. What should the care
b. Place the patient in a Fowler position with the head of of the patient include in the postoperative phase? (Select
the bed at 45 degrees all that apply.)
c. Remove the aspirated fluid and do not reinstill the a Soft diet after 24 hours
gastric aspirate b.Fluid restriction for the first 48 hours because the gums
d. Dilute the gastric tube feeding solution with water and are swollen and painful
continue the feeding c. External heat by pad or compress to hasten the
resolution of the inflammatory swelling
13. The nurse is inserting a sump tube in a patient with d. Warm saline mouthwashes every 2 hours while awake
Crohn's disease who is suspected of having a bowel e. Gargle with peroxide and saline every 4 hours
obstruction. What does the nurse understand is the
benefit of the gastric (Salem) sump tube in comparison to 4. A patient reports an inflamed salivary gland below the
some of the other tubes? right ear. The nurse documents probable inflammation of
a. The tube is radiopaque which gland?
b. The tube is shorter a. The buccal gland
c. The tube is less expensive b. The parotid gland
d. The tube can be connected to suction and others c. The sublingual gland
cannot d. The submandibular gland
14. The nurse is inserting a nasogastric tube for a patient
with pancreatitis. What intervention can the nurse 5. An older adult patient who has been living at home
provide to allow facilitation of the tube insertion? alone is diagnosed with parotitis. What causative bacteria
a. Spray the oropharynx with an anesthetic spray does the nurse suspect is the cause of the parotitis?
b. Have the patient maintain a backward tilt head position a. Methicillin-resistant Streptococcus aureus (MRSA)
c. Allow the patient to sip water as the tube is being b. Pneumococcus
inserted c. Staphylococcus aureus
d. Have the patient eat a cracker as the tube is being d. Streptococcus viridans
inserted
6. A nurse inspects the Stensen duct of the parotid gland
15. The nurse is inserting a nasogastric tube and the to determine inflammation and possible obstruction.
patient begins coughing and is unable to speak. What What area in the oral cavity would the nurse assess?
does the nurse suspect has occurred? a. Buccal mucosa next to the upper molars
a. The nurse has inserted a tube that is too large for the b. Dorsum of the tongue
patient c. Roof of the mouth next to the incisors
b. The nurse has inadvertently inserted the tube into the d. Posterior segment of the tongue near the uvula
trachea
c. This is a normal occurrence and the tube should be left 7. The nurse is obtaining a history on a patient who comes
in place to the clinic. What symptom described by the patient is
d. The tube is most likely defective and should be one of the first symptoms associated with esophageal
immediately removed disease?
a. Dysphagia
CHAPTER 45 b. Malnutrition
1. The nurse is performing an assessment for a patient c. Pain
who presents to the clinic with a lip lesion. The lesion is d. Regurgitation of food
erythemic, is fissuring, and has white hyperkeratosis.
22
d. Hairy leukoplakia

14. A patient comes to the clinic reporting a sore throat.


8. A patient tells the nurse that it feels like food is When assessing the patient, the nurse observes a
"sticking" in the lower portion of the esophagus. What reddened ulcerated lesion on the lip that the patient
motility disorder does the nurse suspect these symptoms states has been there for a couple of weeks but is painless.
indicate? What should the nurse consult with the healthcare
a. Achalasia provider about regarding testing?
b. Diffuse spasm a. HIV
c. Gastroesophageal reflex disease d. Hiatal hernia b. Syphilis
c. Gonorrhea
9. A patient is brought to the emergency department by a d. Herpes simplex
family member, who states that the patient "drank drain
cleaner." What intervention does the nurse anticipate 15. A patient has been taking a 10-day course of
providing to treat this patient? (Select all that apply.) antibiotics for pneumonia and now reports white patches
a. Administering an irritant that will stimulate vomiting that look like milk curds in the mouth. What treatment will
b. Aspirating secretions from the pharynx if respirations the nurse educate the patient about?
are affected a. Nystatin
c. Neutralizing the chemical b. Cephalexin
d. Washing the esophagus with large volumes of water c. Fluocinolone acetonide oral base gel
e. Administering activated charcoal d. Acyclovir

10. The nurse is caring for a patient with a radica Deck


dissection and observes serosanguineout secretions in CHAPTER 46
the wound suction unit during the first postoperative day. 1. A patient has been diagnosed with acute gastritis and
What amount of drainage in the wound unit should the asks the nurse what could have caused it. What is the best
nurse expect to observe? response by the nurse? (Select all that apply.)
a. Between 40 and 80 mL a. "It can be caused by ingestion of strong acids"
b. Approximately 80 to 120 mL b. "You may have ingested some irritating foods"
c. Between 120 and 160 ml (c. 'Is it possible that you are overusing aspirin"
d. Greater than 160 ml d. "It is a hereditary disease"
e. "It is probably your nerves"
11. A patient describes a burning sensation in the
esophagus, pain when swallowing, and frequent 2. The nurse is caring for a patient who has been
indigestion. What does the nurse suspect that these diagnosed with gastritis. To promote fluid balance when
clinical manifestations indicate? treating gastritis, the nurse determines what minimal daily
a. This is an indication that the patient has peptic ulcer intake of fluids is required?
disease a. 1.0 L
b. This is an indication that the patient may have b. 1.5 L
esophageal 'cancer c. 2.0 L
c. These symptoms indicate gastrosophageal reflux d. 2.5 L
disease
d. The clinical manifestations indicate diverticulitis 3. The nurse is preparing to administer medications to a
patient with acute gastritis. The patient has a new
12. A patient has been diagnosed with Zenker prescription for a hista-mine-2 receptor antagonist. Which
diverticulum. What treatment does the nurse anticipate medication will the nurse administer?
educating the patient about? a. Omeprazole
a. A low-residue diet b. Famotidine
b. Chemotherapeutic agents c. Lansoprazole
c. Radiation therapy d. Bismuth salts
d. Surgical removal of the diverticulum
4. A patient comes to the clinic stating, "I think I have an
13. A patient who has positive human immune deficiency ulcer." What is a characteristic associated with peptic
virus (HIV) comes to the clinic reporting white patches ulcer pain that the nurse should inquire about?
with rough hairlike projections on the tongue. The nurse (Select all that apply.)
observes the lesions on the lateral border of the tongue. a. Burning sensation localized in the back or mid-
What abnormality of the mouth does the nurse epigastrium
determine these lesions are? b. Feeling of emptiness that precedes meals from 1 to 3
a. Aphthous stomatitis hours
b. Nicotine stomatitis c. Severe gnawing pain that increases in severity as the
c. Erythroplakia day progresses
23
d. Pain that radiates to the shoulder or jaw c. Gastric penetration
e. Vomiting without associated nausea d. Perforation of the peptic ulcer
5. The nurse is educating a patient about discharge
medications. When should the nurse instruct the patient 11. A patient taking metronidazole for the treatment of H.
to take the antacid medication? pylori states that the medication is causing nausea. What
a. With the meal suggestion can the nurse provide to the patient to
b. 30 minutes before the meal alleviate this problem?
c. 1 to 3 hours after the meal a. Discontinue the use of the medication
d. Immediately after the meal b. Tell the patient to ask the healthcare provider to
prescribe another type of antibiotic
6. A patient is scheduled for a Billroth I procedure for c. Take the medication with meals to decrease the nausea
ulcer management. What does the nurse understand will d. Crush the medication and put it in applesauce
occur when this procedure is performed?
a. A partial gastrectomy is performed with anastomosis of 12. The nurse is educating a patient with peptic ulcer
the stomach segment to the duodenum disease about the disease process. What decreases the
b. A sectioned portion of the stomach is joined to the secretion of bicarbonate from the pancreas into the
jejunum duodenum, resulting in increased acidity of the
c. The antral portion of the stomach is removed and a duodenum?'
vagotomy is performed a. Smoking
d. The vagus nerve is cut and gastric drainage is b. Eating spicy foods
established c. Drinking carbonated beverages
d. Taking antacids
7. The nurse is developing a plan of care for a patient with
peptic ulcer disease. What nursing interventions should 13. The nurse is educating a patient with peptic ulcer
be included in the care plan? (Select all that apply.) about dietary modification. What should the nurse be sure
a. Making neurovascular checks every 4 hours to include when educating this patient?
b. Frequently monitoring hemoglobin and hematocrit a. Avoid extremes of temperature in food and beverages
levels b. Increase the fiber content in the diet
(c. Observing stools and vomitus for color, consistency, c. Decrease the amount of fluid the patient is drinking
and volume d. Increase the protein content in the diet
d. Checking the blood pressure and pulse rate every 15 to
20 minutes 14. The nurse determines that a patient is at risk for the
e. Inserting an indwelling catheter for incontinence development of gastric cancer. What foods should the
nurse encourage the patient to avoid? (Select all that
8. The nurse is caring for a patient who is suspected to apply.)
have developed a peptic ulcer hem-orrhage. Which action a. Fruits
would the nurse perform first? b. Vegetables ) Smoked foods d Pickled foods
a. Place the patient in a recumbent position with the legs b. A reaction to the medication given for the ulcer
elevated c. Gastric penetration
b. Prepare a peripheral and central line for intravenous d. Perforation of the peptic ulcer
infusion e. Whole grains
c. Assess vital signs
d. Call the physician 15. A patient is having bile reflux after gastric surgery and
having the pylorus removed. What pharmacologic therapy
9. A patient sustained second- and third-degree burns should the nurse educate the patient regarding?
over 30% of the body surface area approximately 72 a. The administration of capecitabine
hours ago. What type of ulcer should the nurse be alert b. The administration of omeprazole
for while caring for this patient? c. The administration of famotidine
a. Curling ulcer a. The administration of cholestyramine
b. Peptic ulcer
c. Esophageal ulcer
d. Meckel ulcer

10. A patient is in the hospital for the treatment of peptic


ulcer disease. The nurse finds the patient with a board-
like abdomen, vomiting, and reports of a sudden severe
pain in the abdomen. What does the nurse suspect these
symptoms indicate?
a. The treatment for the peptic ulcer is ineffective
b. A reaction to the medication given for the ulcer

24
CHAPTER 47 6. A patient with irritable bowel syndrome has been
1. The nurse is assessing a patient with appendi-citis. The having more frequent symptoms lately and is not sure
nurse is attempting to elicit a Rovs-ing sign. Where should what lifestyle changes may have occurred. What
the nurse palpate for this indicator of acute appendicitis? suggestion can the nurse provide to identify a trigger for
a. Right lower quadrant the symptoms?
b. Left lower quadrant a. Document how much fluid is being taken to determine if
c. Right upper quadrant the patient is overhydrating
d. Left upper quadrant b. Discontinue the use of any medication presently being
taken to determine if medication is a trigger
2. 2. A patient is not having daily bowel move. ments and c. Begin an exercise regimen and biofeedback to
has begun taking a laxative for this problem. What should determine if external stress is a trigger.
the nurse educate the patient about regarding laxative d. Keep a 1- to 2-week symptom and food diary to identify
use? food triggers
a. When taking the laxatives, plenty of fluids should be
taken as well 7. The nurse is caring for a patient who has malabsorption
b. The laxatives should be taken no more than three syndrome with an undetermined cause. What procedure
times a week or laxative addic. tion will result will the nurse assist with that is the best diagnostic test for
c. Laxatives should not be routinely taken due to this illness?
destruction of nerve endings in the colon a. Ultrasound
d. Laxatives should never be the first response for the b. Endoscopy with mucosal biopsy
treatment of constipation; natural methods should be c. Stool specimen for ova and parasites
employed first d. Pancreatic function tests

3. A patient is admitted to the hospital after not having 8.A patient arrives in the emergency department reporting
had a bowel movement in several days. The nurse right lower abdominal pain that began 4 hours ago and is
observes the patient is having small liquid stools, a grossly getting worse.
distended abdo-men, and abdominal cramping. What The nurse assesses rebound tenderness at McBurney
com. plication can this patient develop related to this point. What does this assessment data indicate to the
problem? nurse?
a. Appendicitis a. Crohn's disease
b. Rectal fissures b. Ulcerative colitis
c. Bowel perforation n c. Appendicitis
d. Diverticulitis d. Diverticulitis

4. The nurse is performing an abdominal assessment for a 9. The nurse is caring for a patient who has had an
patient with diarrhea and auscul-tates a loud rumbling appendectomy. What is the best position for the nurse to
sound in the left lower quadrant. What will the nurse maintain the patient in after the surgery?
document this sound as on the nurse's notes? a. Prone
a. Loud bowel sounds b. Sims' left lateral
b. Borborygmus c. High Fowler
c. Tenesmus d. Supine with head of bed elevated 15 degrees
d. Peristalsis
10. A patient is suspected to have diverticulosis without
5. The nurse is caring for an older adult patient symptoms of diverticulitis. What diagnostic test does the
experiencing fecal incontinence. When planning the care nurse anticipate educating the patient about prior to
of this patient, what should the nurse designate as a scheduling?
priority goal? a. Colonoscopy
a. Maintaining skin integrity b. Barium enema
b. Beginning a bowel program to establish continence c. Flexible sigmoidoscopy
c. Instituting a diet high in fiber and increase fluid intake d. CT scan
d. Determining the need for surgical intervention to
correct the problem 11. The nurse is admitting a patient with a diagnosis of
diverticulitis and assesses that the patient has a board-like
abdomen, no bowel sounds, and reports severe abdominal
pain. What is the nurse's first action?
a. Start an IV with lactated Ringer solution
b. Notify the healthcare provider
c. Administer a retention enema
25
d. Administer an opioid analgesic

12. The nurse is assigned to care for a patient 2 days after


an appendectomy due to a ruptured appendix with
resultant peritonitis. The nurse has just assisted the
patient with ambu-lation to the bedside commode when
the patient points to the surgical site and informs the
nurse that "something gave way." What does the nurse
suspect may have occurred?
a. A drain may have become dislodged
b. Wound dehiscence has occurred
c. Infection has developed
d. The surgical wound has begun to bleed

13. A patient is having a diagnostic workup for reports of


frequent diarrhea, right lower abdominal pain, and
weight loss. The nurse is reviewing the results of the
barium study and notes the presence of "string sign."
What does the nurse recognize that this is significant of?
a. Crohn's disease
b. Ulcerative colitis
c. Irritable bowel syndrome
d. Diverticulitis

14. A patient is being seen in the clinic reporting painful


hemorrhoids. The nurse assesses the patient and
observes the hemorrhoids are prolapsed but able to be
placed back in the rectum manually. The nurse
documents the hemorrhoids as what degree?
a. First degree
b. Second degree
c. Third degree
d. Fourth degree

15. The nurse is irrigating a colostomy when the patient


says,"You will have to stop, I am cramping so badly."
What is the priority action by the nurse?
a. Inform the patient that it will only last a minute and
continue with the procedure
b. Clamp the tubing and give the patient a rest period
c. Stop the irrigation and remove the tube
d. Replace the fluid with cooler water since it is probably
too warm

26

You might also like